Mathematical induction is a mathematical proof technique. It is essentially used to prove that a statement P(n) holds for every natural number n = 0, 1, 2, 3, . . . ; that is, the overall statement is a sequence of infinitely many cases P(0), P(1), P(2), P(3), . . . . Informal metaphors help to explain this technique, such as falling dominoes or climbing a ladder:
Mathematical induction proves that we can climb as high as we like on a ladder, by proving that we can climb onto the bottom rung (the basis) and that from each rung we can climb up to the next one (the step).
A proof by induction consists of two cases. The first, the base case (or basis), proves the statement for n = 0 without assuming any knowledge of other cases. The second case, the induction step, proves that if the statement holds for any given case n = k, then it must also hold for the next case n = k + 1. These two steps establish that the statement holds for every natural number n. The base case does not necessarily begin with n = 0, but often with n = 1, and possibly with any fixed natural number n = N, establishing the truth of the statement for all natural numbers n ≥ N.
The method can be extended to prove statements about more general well-founded structures, such as trees; this generalization, known as structural induction, is used in mathematical logic and computer science. Mathematical induction in this extended sense is closely related to recursion. Mathematical induction is an inference rule used in formal proofs, and in some form is the foundation of all correctness proofs for computer programs.Although its name may suggest otherwise, mathematical induction should not be confused with inductive reasoning as used in philosophy (see Problem of induction). The mathematical method examines infinitely many cases to prove a general statement, but does so by a finite chain of deductive reasoning involving the variable n, which can take infinitely many values.
Homework Statement Homework Equations
With what equations i can express the fact that when dc is aplied to transformer there is mutual inductance only when the voltage is turned on or off and there is no positive feed back represented as an infinite inducyion between the 2 coils?
The Attempt at...
Homework Statement
Using the principle of mathematical induction, prove that for all n>=10, 2^n>n^3
Homework Equations
2^(n+1) = 2(2^n)
(n+1)^3 = n^3 + 3n^2 + 3n +1
The Attempt at a Solution
i) (Base case) Statement is true for n=10
ii)(inductive step) Suppose 2^n > n^3 for some integer >=...
Hey! :o
Let $L$ be a differential operator.
We suppose that we have $n$ equations, that means $\phi: \displaystyle{\bigwedge_{j=1}^n L_j x=f_j}$ and we assume that $\phi$ can be written as $Lx=f \land \psi$, where $\psi$ doesn't contain any $x$.
We prove this by induction on the number of...
Homework Statement
Show that every 2-tree(Definition 3.2) with n internal nodes has n+1 external nodes
Homework Equations
A 2 tree is a tree which nodes have 2 children or no children.
Internal nodes are nodes which have two children
external nodes are leafs.
The Attempt at a Solution
Proof...
Consider a closed electric circuit consisting of a conducting straight rod connected to a DC voltage source switched on. In steady state the current flowing through the conductor creates a steady magnetic field around it with magnetic flux lines that can be represented as concentric circles...
I was thinking about buying these new wireless earbuds called DASH. They are made by a kickstarter company Bragi. They recently told us that in order to connect both earbuds together and also connect it to a device using bluetooth they needed to use magnetic induction. I was wondering what that...
Homework Statement
Prove that \forall n\in \mathbb{N}
\sum\limits_{k=1}^{n}(-1)^{k+1}{n\choose k}\frac{1}{k}=1+\frac{1}{2}+\frac{1}{3}+...+\frac{1}{n}
Homework Equations
-Induction
-Summation
-Binomial coefficient
The Attempt at a Solution
For n=1 equality is true.
For n=m
m-{m\choose...
I'm studying electromagnetic induction and I'm trying to understand a couple of things:
Why is the flux greatest when the angle between the conductor and magnetic field is 90 degrees?
How does the induced e.m.f oppose the change which induces it?
My thoughts:
The flux is greatest when the angle...
Homework Statement
[/B]
Show that the statement holds for all positive integers n
2n ≤ 2^n
Homework Equations
Axiom of induction:
1 ∈ S and
k ∈ S ⇒ k + 1 ∈ S
The Attempt at a Solution
Let S be set of integers
2(1) ≤ 2^1, so S contains 1
k ∈ S,
2k ≤ 2^k
I want to show k + 1 ∈ S,
2k +...
I was just recently reading about Cauchy's proof of the AM-GM inequality by forward backward induction and was simply blown away by it's elegance. However, I have never seen forward backward induction in any of my discrete maths books (including the one devoted just to mathematical induction)...
Homework Statement
Please see the attached file.Homework Equations
$$\frac{dΦ}{dt} = ε$$The Attempt at a Solution
The only way I see is to apply some conservation of energy. But I don't know how. The potential energy is being converted to rotational energy. But how do I find the potential energy?
i have a doubt on this one. we know +ve charge (proton) and neutrons make the most of mass for any matter. Now consider a conductor and through induction, centre of -ve charged is moved toward the object (insulator(static charge on it) with +ve charge say). Now these two attract and conducting...
Homework Statement
Prove that for all natural numbers n, there exists a natural number m^2 such that
n ≤ m^2 ≤ 2n
The Attempt at a Solution
I know how to prove this directly or by construction but my professor wants it solved by induction. When you're solving something by induction you have...
I understand how to construct a proof by induction. I've used it many times, for homework because it was clearly what the book wanted, but when I've tried it in a research setting, it's because I have so little control of the objects in working with. So it has become my impression that since...
Prove that n^5 - 5n^3 + 4n is divisible by 120. for every natural number n greater or equal to 3.
First, i checked if it works for n=3 and it does,
so i could assume it works for some k>=3 so i could write k^5 - 5k^3 + 4k as 120*a a is natural number
so for k+1 i have:
(k+1)^5 - 5(k+1)^3...
Homework Statement
Let A1,A 2,...be an infinite sequence of events such that A1⊂A2⊂... Prove that
Pr(∪Ai)=limn→infPr(An)
∪ is also an iterator that starts from i=1 to infinity. How can you put those iterators?[/B]
Homework Equations
I decided to use induction
The...
I am learning basic motor control and there is a formula list in my book.
The last one is for torque calculation, can anyone point out to me where this one is from or how is it derived?
Hi all,
Assume a standalone synchronous generator feeding power to a single induction machine which is made to run as induction generator at super-synchronous speed without any external electrical load connected to the machine busbars.
What will be the behavior of the induction machine under...
Consider the arrangements shown in figure in which the north pole of a magnet is moved away from a thick conducting loop containing capacitor. Then excess positive charge will arrive on : (Plate B = answer)
Am i correct in marking the side facing the magnet as south pole? ( because of Lenz...
I am new to flux oriented control and i read that this method is to decouple the stator current, one component for flux producing and other for torque,so that so that we can control these two current components to control an AC motor like a DC one(right?)
However when i try to dig into detail...
So I've been wanting to build a particle accelerator for a while, and have kind of been brain storming ideas to make it work. I've been recently trying to figure out how to get the actual acceleration to happen.
I have a few ideas, but the one that I like (assuming it's possible) is using a...
Homework Statement
In the system shown in the picture, there's a current whose constant density is J=0,5A/mm^2. System contains two pieces as shown in the picture, in the area where two pieces intersect, there's no current. If R=1mm and a=1,25mm (a - distance between centers of the circles)...
Hello,
I want to know determining the number of poles by inspecting coils inside the motor. Generally, there are formulas allowing us to plug a frequency, an rpm, and give off the number of poles, or we can just simply look at its nameplate.
My reason for needing the inspecting is that my...
Capacitor-start induction motors use a capacitor in series with the start winding to create torque to start the motor. After the motor gets nearly up to full speed, a switch takes the capacitor out of the circuit. I understand the torque is caused by the phase shift created by having the...
I am just scraping the surface of electromagnetic induction, and I have some questions. First, let me illustrate what I know. I know that in a simple circuit with a battery and a switch, when the switch is turned on, a current flows which produces a magnetic field of strength in proportion to...
Hey folks, I'm looking for a high school laboratory to introduce students to induction. I've searched the usual suspects...Google, YouTube, etc...but nothing has really jumped out. I've ordered some toroids so students can compare the strength of magnetic fields in air versus ferrite, and I'll...
It is observed that the stator current of IM increases on loading. In transformers, the increase in primary current due to loading is significant . Is it same in induction motor? Does the air gap affect it?
Hi all,
I am designing an NFC device but am still a little unsure about the principles behind NFC/RFID. From what I understand:
A primary coil constantly emits a 'carrier frequency' magnetic field.
This field induces an e.m.f. on a passive secondary coil which is in a secondary circuit.
The...
hello guys!
I am confused about determining the equation for the induced emf in a rectangular coil with n turns rotating in a uniform magnetic field.
According to faraday's law of EMI the emf induced in a coil of wire is the rate of change of flux passing through it
$E_{induced}...
Hypothetical, Two parallel wires, wire 1 and wire 2. Trying to understand my system.
A) Wire 1 has a constant current which is being driven by a V and power supply.
Is there a current in wire 2 if the current in Wire 1 is constant?
B) Wires A and B both have current flowing through them. Does...
Homework Statement
I came across these two equations in a chapter on induction motors, and I was wondering if they both calculate the same thing?
##w_r=sw_s##
##w_r=w_s - \frac{p}{2}w##
Homework Equations
The Attempt at a Solution
s is the slip, ##w_r## and ##w_s## are the rotor and...
Let a be a lowering operator and a† be a raising operator.
Prove that a((a†)^n) = n (a†)^(n-1)
Professor suggested to use induction method with formula:
((a†)(a) + [a,a†]) (a†)^(n-1)
But before start applying induction method, I would like to know where the given formula comes from. Someone...
Homework Statement
Use proof by induction to show that 2^n > n^3 for n>9
Homework EquationsThe Attempt at a Solution
My solution is not as required by the question because I cannot really understand the proof by induction. I will give summary of my understanding. Please check my understanding...
In mutual induction, when a coil is turned on or off, it generates voltage in another nearby coil. But where is that voltage coming from? It can't be created, so what form of energy does it convert from?
As far as I know, when the magnetic field cuts across the coil, that generates voltage...
Hi, I'm working through "What is mathematics" by Courant, and in the first chapter he covers proof by mathematical induction. I understand the method, and I do understand the general principle, but I think I'm confused somewhere.
Basically, you have to prove something on the LHS is equal to...
i, I'm stumped on a proof, one problem may be that either I don't know how to deal with exponents of this type, or my algebra went wrong somewhere. It's from the first chapter of Courant's book "What is mathematics" (p.18 q4)
1. Homework Statement
Prove by mathematical induction:
(1 + q)(1 +...
I'm having a bit of trouble wrapping my head around why inductors behave the way they do in certain circuits. Every physical explanation for why ##V = -L\frac{dI}{dt}## that I've seen explains how the voltage across the inductor is developed as the current changes: the changing current means a...
Hi, trying to http://physics.stackexchange.com/questions/189766/teslas-egg-of-columbus# Tesla's egg of Columbus for a science project ().
I have managed to cast a small rod of solid aluminium into a solid egg shaped piece of aluminium. I also have a large glass bowl that could sit inside an...
Homework Statement
Prove that ∑nj=0 n C r = 2nHomework Equations
Defn. of a combination.
Defn. of mathematical induction.The Attempt at a Solution
The formula is true for n=1
2=∑j=01 n C r
= 1 C 0 +1 C 1
=1 + 1
= 2
Now assume that for some k∈ℕ and 0≤ j ≤ k we have
2k = ∑kj=0 (k C j)
Then...
Is the no. of electrons on a brand new unused and untested harddisk is different from no. of electrons on a used hard disk completely filled with data. i mean will there be even a change of one electron?
Please consider the following points before answering: 1- Does all the electrons from a...
I want to known the stator design of an indiction motor. How may turns required for each pole. For exmple if a single phase motors is designed to run at 230v ac, how may turns required in each pole. If the same motor is designed to be run in lower voltage (say 110v ac or 12v ac) what will be the...
Hi everyone...
I have to design a single phase induction motor.
I have some questions and it would be great if you could help me. The parameters I have for the design of the motor are: rated power, rated voltage, frequency, rated speed, power factor, efficiency, full load current and full load...
Hi,
I've seen on on several sites that you can prove that nCr, where r<=n, is a natural number. I'm not sure how to do this by induction.
So I need help on this proof. How do I write this as a mathematical statement at the start of the induction proof?
Thank you
I would appreciate any and all feedback regarding this document currently housed in Google docs. Basically, I generalize induction among natural numbers to an extreme in an environment regarding what I call grammatical systems. Then an induction principle is derived from that which holds in...
Homework Statement
A small circular metal ring of radius r is concentric with a large circular metal ring of radius 10r. Current in the outer ring flows counterclockwise due to an unpictured power supply. By adjusting the power supply, you can adjust I, the current in the large ring. The graph...
Considering the above diagram, I am failing to see why the aluminium ring floats upwards rather than oscillates. I understand Faraday's law, but I am struggling with Lenz's law - as there is an alternating current, an alternating magnetic field is produced, similar to a sine wave. Therefore, I...